what is the solution to this equation -3x=52

Answers

Answer 1

Answer:

-17.3

Step-by-step explanation:

To solve for x, we can divide both sides by -3:

x = -52 / 3 which = -17.3

Answer 2

Answer:

[tex] \boxed{ \boxed{ \bold{ \sf{ - 17.33}}}}[/tex]

Step-by-step explanation:

[tex] \sf{ - 3x = 52}[/tex]

Divide both sides of the equation by -3

⇒[tex] \sf{ \frac{ - 3x}{ - 3} = \frac{52}{ - 3} }[/tex]

Calculate

⇒[tex] \sf{x = - 17.33}[/tex]

Hope I helped!

Best regards!!


Related Questions

Please answer this question now in two minutes

Answers

Answer:

m∠C = 102°

Step-by-step explanation:

This diagram is a Quadrilateral inscribed in a circle

The first step is to determine what m∠B

is

The sum of opposite angles in an inscribed quadrilateral is equal to 180°

m∠D + m∠B = 180°

m∠B = 180° - m∠D

m∠B = 180° - 80°

m∠B = 100°

Second step is we proceed to determine the exterior angles of the circle

m∠ADC = 2 × m∠B

m∠ADC = 2 × 100°

m∠ADC = 200°

m∠ADC = m∠CD + m∠AD

m∠AD = m∠ADC - m∠CD

m∠AD = 200° - 116°

m∠AD = 84°

The third step is to determine m∠BAD

m∠BAD = m∠AD + m∠AB

m∠BAD = 84° + 120°

m∠BAD = 204°

The final step Is to determine what m∠C is

It is important to note that:

m∠BAD is Opposite m∠C

Hence

m∠C = 1/2 × m∠BAD

m∠C = 1/2 × 204

m∠C = 102°

I need help factoring this question, Factor 4(20) + 84.

Answers

Answer:

164

Step-by-step explanation:

B for brackets

O for of

D for division

M for multiplication

A for addition

S for subtraction

You first start with the brackets (20) and multiply with 4 which is equal to 80 and then add it to 84 which makes 164

I hope this helps

what is the answer to 1/8=s-3/4

Answers

Answer:

7/8 =s

Step-by-step explanation:

1/8=s-3/4

Add 3/4

1/8 + 3/4 = s -3/4 +3/4

1/8 + 3/4 = s

Get a common denominator

1/8 + 3/4 *2/2 = s

1/8 + 6/8 =s

7/8 =s

1/8 = s - 3/4

1/8 = s -6/8 ( * 2/2)

7/8 = s

s = 7/8

simpily 2^3×3^2=6^5​

Answers

Answer:

2^3×3^2=6^5​  equation is wrong because

2×2×2×3×3=72

6^5=6×6×6×6×6=36×36×6=7776

the two numbers are not equal

Mate, I think your question is wrong ! ;(

[tex]Corrected \\ Question...\\[/tex] (2^3)^2*(3^2)^3=6^5

Solve the equation for X (If possible please show work)

Answers

Answer:

the correct answer is x=5

How can you change a rational number to a decimal? Can you give an exsample?

Answers

Answer:

1/2=0.5

Step-by-step explanation:

¼=0.25

¾=0.75

Evaluate the following expression. −8 × (−10) −7× 1/−1

Answers

Answer:

87

Step-by-step explanation:

[tex]-8\left(-10\right)-7 \times \frac{1}{-1}=87\\\\\mathrm{Apply\:rule}\:-\left(-a\right)=a\\\\=8\times \:10-7\times \frac{1}{-1}\\\\8\times \:10=80\\\\7\times \frac{1}{-1}=-7\\\\=80-\left(-7\right)\\\\\mathrm{Apply\:rule}\:-\left(-a\right)=a\\\\=80+7\\\\=87[/tex]

solve this equation -2x+9=-5x-15

Answers

Answer:

x = -8

I hope this helps!

The answer would be x=-8

Joey intends to roll a six-sided number cube 100 times. What probability model can he use to predict whether or not each roll will give a result that is divisible by 3?

Answers

Options :

A. Each roll has a 0.117 probability of being divisible by 3.

B. Each roll has a 0.333 probability of being divisible by 3.

C. Each roll has a 0.5 probability of being divisible by 3. D. Each roll has a 0.667 probability of being divisible by 3.

Answer: B. Each roll has a 0.333 probability of being divisible by 3.

Step-by-step explanation:

Sample space for a six-sided number cube :

1, 2, 3, 4, 5, 6

Number of outcomes divisible by 3:

(3, 6) = 2

Probability of an event = Number of required outcomes / total number of possible items

Probability (getting a number divisible by 3):

(Number of outcomes divisible by 3 / total outcomes in sample space)

Probability (getting a number divisible by 3):

2 / 6 = 1/3

= 0.333

need help will give 5 stars.

Answers

Answer:

t=0.64

Step-by-step explanation:

h = -16t^2 +4t +4

We want h =0 since it is hitting the ground

0 = -16t^2 +4t +4

Using the quadratic formula

a = -16  b = 4  c=4

-b ± sqrt( b^2 -4ac)

----------------------------

         2a

-4 ± sqrt( 4^2 -4(-16)4)

----------------------------

         2(-16)

-4 ± sqrt( 16+ 256)

----------------------------

         -32

-4 ± sqrt( 272)

----------------------------

         -32

-4 ± sqrt( 16*17)

----------------------------

         -32

-4 ± sqrt( 16) sqrt(17)

----------------------------

         -32

-4 ± 4 sqrt(17)

----------------------------

         -32

Divide by -4

1 ±  sqrt(17)

----------------------------

         8

To the nearest hundredth

t=-0.39

t=0.64

Since time cannot be negative

t=0.64

Answer:

0.64  

Step-by-step explanation:

0 = -16t^2 + 4t + 4

-4(4t^2 - t -1) = 0

t = [-(-1) +/- sqrt (1 - 4*4*-1)] / 8)

t = 0.64, -0.39

answer is 0.64

The height of a building model is 2% of its actual height. If the building
model is 3 feet tall, how tall is the actual building?

Answers

Answer:

x = 150 feets

Step-by-step explanation:

Given that,

The height of a building model is 2% of its actual height.

The building model is 3 feet tall, h = 3 feet

We need to find the height of the actual building. Let it is x.

According to question,

h = 2% of x

We have, h = 3 feet

So,

[tex]x=\dfrac{h}{2\%}\\\\x=\dfrac{3}{2/100}\\\\x=150\ \text{feet}[/tex]

So, the actual height of the building is 150 feets.

A combination lock uses three numbers between 1 and 46 with​ repetition, and they must be selected in the correct sequence. Is the name of​ "combination lock"​ appropriate? Why or why​ not? Choose the correct answer below. A. ​No, because the multiplication counting rule would be used to determine the total number of combinations. B. ​Yes, because the combinations rule would be used to determine the total number of combinations. C. ​No, because factorials would be used to determine the total number of combinations. D. ​No, because the permutations rule would be used to determine the total number of combinations.

Answers

The correct answer is D. ​No because the permutations rule would be used to determine the total number of combinations.

Explanation:

The difference between a combination and a permutation is that in permutations the order is considered. This applies to the numbers in a lock because these need to be in order. Therefore, to analyze the permutations in a lock, the rule for permutations should be used. This includes the general formula P (n,r) =[tex]\frac{n!}{(n-r) !}[/tex]; in this, n is the number of objects and r refers to the objects used in a permutation. Thus, the term "combination" is inappropriate because this is a permutation, and the permutation rule should be used.

4x=24 solve equation

Answers

Answer:

x=6

Step-by-step explanation:

Rearrange:

Rearrange the equation by subtracting what is to the right of the equal sign from both sides of the equation :

                    4*x-(24)=0

Step by step solution :

STEP

1

:

Pulling out like terms

1.1     Pull out like factors :

  4x - 24  =   4 • (x - 6)

Equation at the end of step

1

:

STEP

2

:

Equations which are never true

2.1      Solve :    4   =  0

This equation has no solution.

A a non-zero constant never equals zero.

Solving a Single Variable Equation:

2.2      Solve  :    x-6 = 0

Add  6  to both sides of the equation :

                     x = 6

One solution was found :

x = 6

Answer:

x= 24/ 4

Step-by-step explanation:

You can simplify it

x= 6/1 which is x= 6

A rectangular prism has a volume of 864 cubic units. How many cubic unit will fill the volume of the solid if they were packed without any gaps or overlaps

Answers

Answer: 864.

Step-by-step explanation:

The volume of a rectangular prism has a volume equal to:

V = W*L*H

W = width

L = length

H = height

We know that the volume is equal to 864 cubic units.

This means that if we want to fill the prism such that there is no gap or overlap, we should use exactly 864 unit cubes.

If 4SINB=3SIN(2A+B) :
Prove that:7COT(A+B)=COTA

Answers

Answer:

Step-by-step explanation:

Given the expression 4sinB = 3sin(2A+B), we are to show that the expression 7cot(A+B) = cotA

Starting with the expression

4sinB= 3sin(2A+B)

Let us re write angle B = (A + B) - A

and 2A + B = (A + B) + A

Substituting the derived expression back into the original expression ww will have;

4Sin{(A + B) - A } = 3Sin{(A + B)+ A}

From trigonometry identity;

Sin(D+E) = SinDcosE + CosDSinE

Sin(D-E) = SinDcosE - CosDSinE

Applying this in the expression above;

4{Sin(A+B)CosA - Cos(A+B)SinA} = 3{Sin(A+B)CosA + Cos(A+B)sinA}

Open the bracket

4Sin(A+B)CosA - 4Cos(A+B)SinA = 3Sin(A+B)CosA + 3Cos(A+B)sinA

Collecting like terms

4Sin(A+B)CosA - 3Sin(A+B)cosA = 3Cos(A+B)sinA + 4Cos(A+B)sinA

Sin(A+B)CosA = 7Cos(A+B)sinA

Divide both sides by sinA

Sin(A+B)CosA/sinA= 7Cos(A+B)sinA/sinA

Since cosA/sinA = cotA, the expression becomes;

Sin(A+B)cotA = 7Cos(A+B)

Finally, divide both sides of the resulting equation by sin(A+B)

Sin(A+B)cotA/sin(A+B) = 7Cos(A+B)/sin(A+B)

CotA = 7cot(A+B) Proved!

A zoo train ride costs $4 per adult and $1 per child. On a certain day, the total number of adults (a) and children (c) who took the ride was 27, and the total money collected was $60. What was the number of children and the number of adults who took the train ride that day, and which pair of equations can be solved to find the numbers? 1) 11 children and 16 adults Equation 1: a + c = 27 Equation 2: 4a + c = 60 2) 16 children and 11 adults Equation 1: a + c = 27 Equation 2: 4a + c = 60 3) 11 children and 16 adults Equation 1: a + c = 27 Equation 2: 4a − c = 60 4) 16 children and 11 adults Equation 1: a + c = 27 Equation 2: 4a − c = 60

Answers

Answer:

11 adults and 16 children

Step-by-step explanation:

a + c = 27 and 4a + c = 60

3a = 60 - 27 = 33

a= 11  

so c = 16

a blue die and a green die are rolled. find the probability that the blue and green are both less than 6​

Answers

Answer

5/6 maybe

Step-by-step explanation:

Solve. 4x−y−2z=−8 −2x+4z=−4 x+2y=6 Enter your answer, in the form (x,y,z), in the boxes in simplest terms. x= y= z=

Answers

Answer:

(-2, 4, 2)

Where x = -2, y = 4, and z = 2.

Step-by-step explanation:

We are given the system of three equations:

[tex]\displaystyle \left\{ \begin{array}{l} 4x -y -2z = -8 \\ -2x + 4z = -4 \\ x + 2y = 6 \end{array}[/tex]

And we want to find the value of each variable.

Note that both the second and third equations have an x.

Therefore, we can isolate the variables for the second and third equation and then substitute them into the first equation to make the first equation all one variable.

Solve the second equation for z:

[tex]\displaystyle \begin{aligned} -2x+4z&=-4 \\ x - 2 &= 2z \\ z&= \frac{x-2}{2}\end{aligned}[/tex]

Likewise, solve the third equation for y:

[tex]\displaystyle \begin{aligned} x+2y &= 6\\ 2y &= 6-x \\ y &= \frac{6-x}{2} \end{aligned}[/tex]

Substitute the above equations into the first:

[tex]\displaystyle 4x - \left(\frac{6-x}{2}\right) - 2\left(\frac{x-2}{2}\right)=-8[/tex]

And solve for x:

[tex]\displaystyle \begin{aligned} 4x+\left(\frac{x-6}{2}\right)+(2-x) &= -8 \\ \\ 8x +(x-6) +(4-2x) &= -16 \\ \\ 7x-2 &= -16 \\ \\ 7x &= -14 \\ \\ x &= -2\end{aligned}[/tex]

Hence, x = -2.

Find z and y using their respective equations:

Second equation:

[tex]\displaystyle \begin{aligned} z&=\frac{x-2}{2} \\ &= \frac{(-2)-2}{2} \\ &= \frac{-4}{2} \\ &= -2\end{aligned}[/tex]

Third equation:

[tex]\displaystyle \begin{aligned} y &= \frac{6-x}{2}\\ &= \frac{6-(-2)}{2}\\ &= \frac{8}{2}\\ &=4\end{aligned}[/tex]

In conclusion, the solution is (-2, 4, -2)

Answer:

x = -2

y =4

z=-2

Step-by-step explanation:

4x−y−2z=−8

−2x+4z=−4

x+2y=6

Solve the second equation for x

x = 6 -2y

Substitute into the first two equations

4x−y−2z=−8

4(6-2y) -y -2 = 8  

24 -8y-y -2z = 8

-9y -2z = -32

−2(6-2y)+4z=−4

-12 +4y +4z = -4

4y+4z = 8

Divide by 4

y+z = 2

z =2-y

Substitute this into -9y -2z = -32

-9y -2(2-y) = -32

-9y -4 +2y = -32

-7y -4 = -32

-7y =-28

y =4

Now find z

z = 2-y

z = 2-4

z = -2

Now find x

x = 6 -2y

x = 6 -2(4)

x =6-8

x = -2

please can someone help me solve this.. please help!!​

Answers

Step-by-step explanation:

Hello,

Firstly just look to triangle BDE,

Here, you will find that,

140° = y+80° {the exterior and opposite interior angle of a triangle is equal}.

or, y= 140°-80° {shifting 80° to another side and subtracting it.}

Therefore, the value of y is 60°.

now, let's simply work with line EB or EG. we get;

angle GEF + y=180° { being a linear pair}.

or, angle GEF + 60°= 180°

or, angle GEF = 180°-60°

Therefore, the value of angle GEF = 120°.

now, looking in triangle EFG, we get;

angle GEF + 35°+x= 180° { the sum of interior angle of a triangle is 180°}.

or, 120°+35°+ x= 180°

or, x= 180°- 155°

Therefore, the value of x is 25°.

now, lastly finding the value of "z"

We find that x= z {being vertical opposite angle}

or, z =25°

Therefore, the value of z is 25°.

So, the values are,

x=25°

y=60°

and z= 25°

Hope it helps...

A bag contains twelve marbles, which includes seven red marbles and five blue marbles. Roja reaches into the bag and pulls out four marbles. a) How many different sets of four marbles can be pulled from this bag? b) How many of these sets contain two red marbles and two blue marbles? c) How many of these sets contain all red marbles? d) How many of these sets contain all red marbles or all blue marbles?

Answers

Answer:

a) 495

b) 210

c) 35

d) 40

Step-by-step explanation:

Given a total of 12 marbles.

n = 12

Number of red marbles = 7

Number of blue marbles = 5

a) Number of different sets of 4 marbles that can be made from this bag ?

This is a simple combination problem.

where n = 12 and r = 4.

So, answer will be:

[tex]_{12}C_4[/tex]

Formula:

[tex]_{n}C_r = \dfrac{n!}{(n-r)!r!}[/tex]

[tex]_{12}C_4 = \dfrac{12!}{(8)!4!} = \dfrac{12\times 11\times 10\times 9}{4 \times 3\times 2} =\bold{495}[/tex]

b) Two red and two blue marbles:

The answer will be:

[tex]_{7}C_2 \times _{5}C_2 = \dfrac{7\times 6}{2} \times \dfrac{5\times 4}{2} =\bold{210}[/tex]

c) all red marbles.(4 chosen out of 7 red and 0 chosen out of 5 blue marbles)

[tex]_{7}C_4 \times _{5}C_0 = \dfrac{7\times 6\times 5\times 4}{4\times 3\times 2} =\bold{35}[/tex]

d) all red or all blue.(all red marbles plus all blue marbles)

All red marbles:

[tex]_{7}C_4 \times _{5}C_0 = \dfrac{7\times 6\times 5\times 4}{4\times 3\times 2} \times 1=\bold{35}[/tex]

All blue marbles:

[tex]_{7}C_0 \times _{5}C_4 = 1 \times \dfrac{5\times 4\times 3\times 2}{4\times 3\times 2} =\bold{5}[/tex]

So, answer is 40.

Evaluate the function below at x=5. Then, enter your solution. f(x)=3(2)^x

Answers

Answer:

Solution: f(5) = 96

Step-by-step explanation:

f(5) = 3(2)^5

f(5) = 3 (2 × 2 × 2 × 2 × 2)

f(5) = 3 (32)

f(5) = 96

Nancy needs at least 1000 gigabytes of storage to take pictures and videos on her upcoming vacation. She
checks and finds that she has 105 GB available on her phone. She plans on buying additional memory
cards to get the rest of the storage she needs.
The cheapest memory cards she can find each hold 256 GB and cost $10. She wants spend as little money
as possible and still get the storage she needs.
Let C represent the number of memory cards that Nancy buys.

Answers

Answer:

C = 4 memory cards.

Step-by-step explanation:

256 × 4 = 1024

1024 + 105 = 1129 GB

She needs 4 memory cards.

Nancy needs to buy 4 memory cards.

Given that Nancy needs at least 1000 gigabytes of storage to take pictures and videos on her upcoming vacation, and she checks and finds that she has 105 GB available on her phone, and she plans on buying additional memory cards to get the rest of the storage she needs, and the cheapest memory cards she can find each hold 256 GB and cost $ 10, and she wants to spend as little money as possible and still get the storage she needs, to determine how many memory cards to buy, the following calculation must be performed:

(1000 - 105) / 256 = C 895/256 = C 3.49 = C

So if Nancy buys 3 cards she will still be short on gigabytes. Therefore, she must buy 4 memory cards.

Learn more in https://brainly.com/question/9154717

Can someone help me with this please it’s algebra 2

Answers

Answer:

7 8 9

Step-by-step explanation:

If sin Θ = 5 over 6, what are the values of cos Θ and tan Θ?

Answers

Answer:

Check explanation

Step-by-step explanation:

Sin∅=5/6

Opp=5. Hyp=6

Adj= (√6²+5²)

= √11

Cos∅=(√11)/6

Tan∅=5/(√11)

Answer: cos(theta) = sqrt(11)/6 ; tan(theta)=5/sqrt(11).

Explanation:
sin inverse is restricted to the first and fourth quadrant, so the other side of the triangle is positive sqrt(11) by the Pythagorean theorem.

determine the image of the point p[-3,10) under the translation [5,-7]

Answers

[tex](-3+5,10-7)=(2,3)[/tex]

Need help please asap this is not asap but please still give an answer im stuck

Answers

Answer:

135 cubes

Step-by-step explanation:

First, find the volume of the box with the equation V = Bh, where B is the area of the base and h is the height.

V = (2.25)(0.75)(1.25)

V = 2.109375

Next, find the volume of one cube with the side length 1/4 with V = Bh:

V = (0.25)(0.25)(0.25)

V = 0.015625

Then, divide the volume of the box by the volume of one cube:

2.109375 / 0.015625

= 135

Solve for x in the equation x squared + 11 x + StartFraction 121 Over 4 EndFraction = StartFraction 125 Over 4 EndFraction.

Answers

Answer:

Below

Step-by-step explanation:

● x^2 + 11x + 121/4 = 125/4

Substract 125/4 from both sides:

● x^2 + 11x + 121/4-125/4= 125/4 -125/4

● x^2 + 11x - (-4/4) = 0

● x^2 +11x -(-1) = 0

● x^2 + 11 x + 1 = 0

This is a quadratic equation so we will use the determinanant (b^2-4ac)

● a = 1

● b = 11

● c = 1

● b^2-4ac = 11^2-4*1*1 = 117

So this equation has two solutions:

● x = (-b -/+ √(b^2-4ac) ) / 2a

● x = (-11 -/+ √(117) ) / 2

● x = (-11 -/+ 3√(13))/ 2

● x = -0.91 or x = -10.9

Round to the nearest unit

● x = -1 or x = -11

The solutions are { -1,-11}

The solution of the equation x² + 11x + (121/4) = 125/4 will be 0.09 and negative 11.09.

What is the solution to the equation?

The distribution of weights to the variables involved that establishes the equilibrium in the calculation is referred to as a result.

The equation is given below.

x² + 11x + (121/4) = 125/4

Simplify the equation, then the equation will be

4x² + 44x + 121 = 125

4x² + 44x + 121 - 125 = 0

4x² + 44x - 4 = 0

x² + 11x - 1 = 0

We know that the formula, then we have

[tex]\rm x = \dfrac{-b \pm \sqrt {b^2 - 4ac}}{2a}[/tex]

The value of a = 1, b = 11, and c = -1. Then we have

[tex]\rm x = \dfrac{-11 \pm \sqrt {11^2 - 4 \times 1 \times (-1)}}{2 \times 1}\\\rm x = \dfrac{-11 \pm \sqrt {121 +4}}{2 }\\x = \dfrac{-11 \pm \sqrt {125}}{2 }[/tex]

Simplify the equation, then we have

x = (- 11 ± 11.18) / 2

x = (-11 - 11.18) / 2, (-11 + 11.18) / 2

x = -11.09, 0.09

The solution of the equation will be 0.09 and negative 11.09.

More about the solution of the equation link is given below.

https://brainly.com/question/545403

#SPJ6

Question 3 of 10
True or false? In a two-column proof, the right column states your reasons.
A. True
OB. False
SUBMIT

Answers

Answer:

A- True

Step-by-step explanation:

If you search a picture of the graph then you will see it as well!!! Hope this helps!!!!

PLEASE HELP, WILL GIVE BRAINLIEST IF CORRECT!!!! (08.06 MC) Mike and his friends bought cheese wafers for $2 per packet and chocolate wafers for $1 per packet at a carnival. They spent a total of $25 to buy a total of 20 packets of wafers of the two varieties. Part A: Write a system of equations that can be solved to find the number of packets of cheese wafers and the number of packets of chocolate wafers that Mike and his friends bought at the carnival. Define the variables used in the equations. (5 points) Part B: How many packets of chocolate wafers and cheese wafers did they buy? Explain how you got the answer and why you selected a particular method to get the answer. (5 points)

Answers

Answer:

x = 5 , y = 15

Step-by-step explanation:

You can solve this using substitution.

Let the quantity of cheese wafers be denoted by x and the quantity of chocolate wafers denoted by y

2x + 1y = 25

x + y = 20

These two equations are the answer to part A, (remember to include the above prompt which says what x and y denote).

For part B I used substitution because it was more applicable to the question then addition or elimination.

ACTUAL WORK

Set 2x + 1y = 25 equal to x

x = 25 - y / 2

Replace x with y in the second equation

(25 - y / 2) + y = 20

And solve for y

y = 15

Since we know what y is we can replace y in the second equation and find what x is

x + 15 = 20

Solve for x

x = 5

Answer:

5 Cheese Wafers and 15 Chocolate Wafers

Step-by-step explanation:

Victor fue al mercado para comprar manzanas, naranjas y platanos; las naranjas costaron el doble de lo 1ue pago por las manzanas y los platanos costaron 8 pesos menos que pas manzanas, en total gasto 100 pesos. Determina el precio de las manzanas, naranjas y platanos

Answers

Answer:

El precio de las manzanas = 27 pesos

El precio de las naranjas = 54 pesos

El precio de las bananas = 19 pesos

Step-by-step explanation:

Los parámetros dados son;

El monto total gastado = 100 pesos

Sea el precio de las naranjas = x

Sea el precio de las manzanas = y

Sea el precio de los plátanos = z

La cantidad pagada por las naranjas = 2 · y = x

La cantidad pagada por los plátanos = y - 8 = z

Por lo tanto, tenemos;

La cantidad total gastada = La cantidad pagada por las naranjas + La cantidad pagada por las bananas + La cantidad pagada por las manzanas

∴ El monto total gastado = 100 pesos = 2 · y + y - 8 + y

100 = 4 · años - 8

4 · y = 100 + 8 = 108

y = 108/4 = 27

y = 27

De

z = y - 8 tenemos;

z = 27 - 8 = 19

De 2 · y = x, tenemos;

2 × 27 = x

x = 54

Por lo tanto;

El precio de las naranjas = 54 pesos

El precio de las manzanas = 27 pesos

El precio de los plátanos = 19 pesos.

Other Questions
Which value would complete the last cell?(1 point)3.0100.025.04.0 The ratio of boys to girls in the ninth grade is 7 to 9. there are 218 girls set up a proportion to model this information TB MC Qu. 8-174 LBC Corporation makes and sells ... LBC Corporation makes and sells a product called Product WZ. Each unit of Product WZ requires 2.0 hours of direct labor at the rate of $16.00 per direct labor-hour. Management would like you to prepare a Direct Labor Budget for June. The company plans to sell 39,000 units of Product WZ in June. The finished goods inventories on June 1 and June 30 are budgeted to be 610 and 110 units, respectively. Budgeted direct labor costs for June would be: Consider the circle of radius 10 centered at the origin. Find an equation of the line tangent to the circle at the point (6, 8) is 1+isqrt3 a complex number Menlo Company distributes a single product. The companys sales and expenses for last month follow: Total Per unitSales $314,000 $20Variable expenses 219,800 14Contribution margin 94,200 6 Fixed expenses 75,000Net operating income 19,200Required: a. What is the monthly break-even point in unit sales and in dollar sales? b. Without resorting to computations, what is the total contribution margin at the break-even point? c. How many units would have to be sold each month to attain a target profit of S27,600? d. Verify your answer by preparing a contribution format income statement at the target sales level. e. Refer to the original data. Compute the company's margin of safety in both dollar and percentage terms. f. What is the company's CM ratio? If sales increase by $76,000 per month and there is no change in fixed expenses, by how much would you expect monthly net operating income to increase? In a(n) __________, the subject comes before the verb.(1 point) 1 singular noun 2 standard sentence 3 inverted sentence 4 prepositional phrase "A customer in the 28% tax bracket has $4,000 of capital gains and $12,000 of capital losses. How much unused loss is carried forward to the next tax year?" An aluminum rod 17.400 cm long at 20C is heated to 100C. What is its new length? Aluminum has a linear expansion coefficient of 25 10-6 C-1. Solve for x: x/25 > 5 Use the map as a reference and select the best answer. In ____ the cataumbo lightning storms last for about 10 hours! La hermana de Sofa es ________. A) Sofa El padre Leonor es ________. B) Letizia La madre Leonor y Sofa es ________. C) Leonor La abuela de Leonor y Sofa es ________. D) Felipe IV Unprecedented industrial growth in the country of Remo has created serious environmental problems because factories there lack adequate pollution control systems. Remo is developing a clean growth plan that includes environmental regulations that will require the installation of such systems. Since no companies in Remo currently produce pollution-control systems, the plan, if implemented, will create significant opportunities for foreign exporters to market pollution-control systemsWhich of the following, if true, most seriously weakens the argument?A. The clean growth plan will provide tax incentives for local business to develop and manufacture pollution-control systemsB. Foreign exporters would provide factory-trained technicians to maintain the pollution-control systems sold in Remo.C. Industrial lobbyists sponsored by local business in Remo are trying to prevent the implementation of the government regulationsD. The regulations that Remo plans to implement are much less strict than those in neighboring nations.E. Pollution in Remo has caused serious health problems for workers, contributing to a significant increase in the number of workdays lost to illness The Constitution states limits on the powers of the national government. What type of governmental system does this reflect When you encounter a T-critical value of 2.50, which T-obtained value would reject the null hypothesis? A) Tobt = 2.75 B) Tobt = -2.75 C) Tobt = 2.40 D) Tobt = - 2.40 Wyckam Manufacturing Inc. has provided the following information concerning its manufacturing costs: Fixed Cost per Month Cost per Machine-Hour Direct materials $ 5.40 Direct labor $ 42,400 Supplies $ 0.30 Utilities $ 1,700 $ 0.25 Depreciation $ 15,200 Insurance $ 11,600 For example, utilities should be $1,700 per month plus $0.25 per machine-hour. The company expects to work 4,200 machine-hours in June. Note that the companys direct labor is a fixed cost.Required:Prepare the company's planning budget for manufacturing costs for June. Water discharging into a 10-m-wide rectangular horizontal channel from a sluice gate is observed to have undergone a hydraulic jump. The flow depth and velocity before the jump are 0.8m and 7m/s, respectively. Determine (a) the flow depth and the Froude number after the jump (b) the head loss (c) the dissipation ratio. Gavin goes to the market and buys one rectangle shaped board. The length of the board is 16 cm and width of board is 10 cm. If he wants to add a 2 cm wooden border around the board, what will be the area of the rectangle board? Analyze the following scenarios to determine who can appropriately access health information.1. Mrs. John Smith is requesting the emergency room records from last week of her daughter, Katy. Mrs. Smith is the noncustodial parent of Katy, who lives with her dad. Should you release the records to her? Why or why not?2. Mr. Fred Mitchell is requesting the birth record for Amy, his birth daughter. Mr. and Mrs. Mitchell gave Amy up for adoption four years ago. Should you release the records to him? Why or why not?3. Mrs. Lynn Olsen is requesting the lab results of her husband, Tim. She has a note. signed by him, giving his permission for her to have the records. Should you release the records to her? Why or Why not?4. An investigator from the Health and Human Services department is conducting an audit of patient records and has provided a list of records that they want to review. Should you release the information to the investigator? Why or why not?5. Dr. Rex Harrisson is requesting the medical records of Martha Flynn. He states he is a family friend and has been asked by Mrs. Flynn's son to review her last inpatient admission for appropriateness of care. Should you release the records to Dr. Harrison? Why or why not? The width of a rectangle measures (8.3c-8.4d)(8.3c8.4d) centimeters, and its length measures (5.3c+4.8d)(5.3c+4.8d) centimeters. Which expression represents the perimeter, in centimeters, of the rectangle?